Analyzing a Smooth Curve for -π < t < π

  • Thread starter Thread starter Masgr404
  • Start date Start date
  • Tags Tags
    Curve Smooth
Click For Summary
The discussion focuses on determining the smoothness of the curve defined by R(t) = (4sin^3(t), 4cos^3(t)) for -π < t < π. Participants clarify that while sine and cosine functions are inherently smooth, the smoothness of R(t) must be assessed by examining its derivatives. Concerns are raised about points where the denominator of any derived fraction could approach zero, potentially indicating discontinuities. The definition of a smooth curve is reiterated, emphasizing that it should not have any abrupt changes or corners. Overall, the thread seeks to clarify the conditions under which R(t) remains a smooth curve within the specified interval.
Masgr404
Messages
2
Reaction score
0

Homework Statement



Determine where r(t) is a smooth curve for -pi <t<pi
R(t)= (x(t),y(t))=(4sin^3(t), 4cos^3(t))

Homework Equations





The Attempt at a Solution



To be honest I have no idea where to start. I know what a smooth function is but my understanding is that the sin(t) and cos(t) functions over all of t are smooth. No corners.
Any starting help would be appreciated.
 
Physics news on Phys.org
Yes, but you are NOT asked if sine and cosine are smooth- you are asked if F is smooth. What happens if the denominator of a fraction goes to 0? What fraction is involved here?
 
The function is not continuous at that particular point that makes the denominator go to zero.
Perhaps we could rewrite in the complex plane?
 
No, it is not necessary to work with the complex plane. What is the definition of "smooth curve"?
 
As someone on mathstackexchange said, a smooth curve is a curve with no stubble, like this: :bugeye:
 

Attachments

  • smoothness.jpg
    smoothness.jpg
    15.2 KB · Views: 440
Last edited:
Question: A clock's minute hand has length 4 and its hour hand has length 3. What is the distance between the tips at the moment when it is increasing most rapidly?(Putnam Exam Question) Answer: Making assumption that both the hands moves at constant angular velocities, the answer is ## \sqrt{7} .## But don't you think this assumption is somewhat doubtful and wrong?

Similar threads

Replies
2
Views
2K
  • · Replies 1 ·
Replies
1
Views
2K
  • · Replies 2 ·
Replies
2
Views
2K
  • · Replies 8 ·
Replies
8
Views
2K
  • · Replies 1 ·
Replies
1
Views
1K
Replies
3
Views
2K
  • · Replies 4 ·
Replies
4
Views
2K
  • · Replies 2 ·
Replies
2
Views
2K
  • · Replies 1 ·
Replies
1
Views
2K
  • · Replies 10 ·
Replies
10
Views
2K